Re: [obm-l] Problema de Desigualdade

2015-06-11 Por tôpico Douglas Oliveira de Lima
Então não é trabalhoso, mas (a/b)^2 = 1 + a/b - b/a não deveria ser
provado?
Desenvolvendo da pra ver que é, neste caso tem mais conta pra fazer.

Forte abraço
Douglas Oliveira.



Em 10 de junho de 2015 12:00, Alexandre Antunes 
prof.alexandreantu...@gmail.com escreveu:


 Bom dia,

 Estou no trabalho, mas vou arriscar a minha primeira resposta no grupo.

 Desenvolvi os dois lados da expressao.

 (a/b)^2 + (b/c)^2 + (c/a)^2 = 3 + (a/b + b/c + c/a) - (b/a + c/b + a/c)

 Como (a/b)^2 = 1 + a/b - b/a
 O mesmo para os demais termos

 Fica provado a proposição.

 O que acham desse trabalhoso caminho?!?!
 Em 10/06/2015 09:00, Pacini Bores pacini.bo...@globo.com escreveu:

 Ok Mariana.

 Abraços

 Pacini

 Em 9 de junho de 2015 21:11, Mariana Groff 
 bigolingroff.mari...@gmail.com escreveu:

 Oi Pacini,
 Fiz do seguinte modo:
 f (x)=x^2-x+1/x=1 = x^3-x^2+1=x = x^3-x^2-x+1=0 =x^2
 (x-1)-(x-1)=0 = (x^2-1)(x-1)=0
 O que podemos ver que é verdade, analisando ambos os casos: em que x=1
 e o caso em que 0 x 1.
 Abraços,
 Mariana
  Em 09/06/2015 20:55, Pacini Bores pacini.bo...@globo.com escreveu:

 Oi Mariana,

 Determinei o mínimo da função usando a derivada. Não entendi o seu
 caminho, pois a função é

 f(x) = x^2-x+1/x.

 Abraços

 Pacini

 Em 9 de junho de 2015 18:09, Mariana Groff 
 bigolingroff.mari...@gmail.com escreveu:

Oi Pacini,
 Compreendi seu raciocínio. Para provar que f(x)=1, basta analisarmos
 que (x^2-1)(x-1)=0, o que verifica-se pois se x=1, o produto é 
 claramente
 não-negativo e se 0x1, vemos que, tanto x^2-1 quanto x-1 são negativos,
 tornando o produto positivo, isso?


 Em 9 de junho de 2015 11:48, Pacini Bores pacini.bo...@globo.com
 escreveu:

 Oi Mariana,
  Observe que provar  a desigualdade pedida  é equivalente  provar que
 :

 {(a/b)^2-a/b+b/a} + {(b/c)^2-b/c+c/b} +{(c/a)^2-c/a+a/c} =3, ok ?

 Agora façamos o seguinte :

 Seja f(x)= x^2-x+1/x, verifique que para x0 o valor mínimo de f é 1.

 Donde teremos a desigualdade provada.

  Estou certo pessoal ?

 Abraços

 Pacini


 Em 8 de junho de 2015 20:30, Raphael Aureliano raphael0...@gmail.com
  escreveu:

 Ah não, desculpa, errei em Cauchy ...

 Att.
 Raphael
 Em 08/06/2015 20:27, Raphael Aureliano raphael0...@gmail.com
 escreveu:

 MA=MG
 LE=(a/b+b/c+c/a)^2=(3cbrt(abc/abc))^2 =9

 Por Cauchy
 LD=(a+b+c)(1/a+1/b+1/c)=(sqrt(a/a) +sqrt(b/b)+sqrt(c/c))^2 =9

 LE=9=LD
  Em 08/06/2015 19:20, Mariana Groff 
 bigolingroff.mari...@gmail.com escreveu:

 Boa Noite,

 (British Mathematical Olympiad - Round 2 - 2005)
 Sejam a,b e c reais positivos.
 Prove que

 (a/b+b/c+c/a)^2=(a+b+c)(1/a+1/b+1/c)

 Atenciosamente,
 Mariana

 --
 Esta mensagem foi verificada pelo sistema de antivírus e
 acredita-se estar livre de perigo.


 --
 Esta mensagem foi verificada pelo sistema de antivírus e
 acredita-se estar livre de perigo.



 --
 Esta mensagem foi verificada pelo sistema de antivírus e
 acredita-se estar livre de perigo.



 --
 Esta mensagem foi verificada pelo sistema de antivírus e
 acredita-se estar livre de perigo.



 --
 Esta mensagem foi verificada pelo sistema de antivírus e
 acredita-se estar livre de perigo.


 --
 Esta mensagem foi verificada pelo sistema de antivírus e
 acredita-se estar livre de perigo.



 --
 Esta mensagem foi verificada pelo sistema de antivírus e
 acredita-se estar livre de perigo.


 --
 Esta mensagem foi verificada pelo sistema de antivírus e
 acredita-se estar livre de perigo.


-- 
Esta mensagem foi verificada pelo sistema de antiv�rus e
 acredita-se estar livre de perigo.



Re: [obm-l] Problema de Desigualdade

2015-06-11 Por tôpico Alexandre Antunes
Boa tarde,

Pensei em fazer essa prova por indução ... Ainda não consegui parar para
finalizar.

Achei que era um caminho possível!!!
Em 11/06/2015 14:28, Douglas Oliveira de Lima 
profdouglaso.del...@gmail.com escreveu:

 Então não é trabalhoso, mas (a/b)^2 = 1 + a/b - b/a não deveria ser
 provado?
 Desenvolvendo da pra ver que é, neste caso tem mais conta pra fazer.

 Forte abraço
 Douglas Oliveira.



 Em 10 de junho de 2015 12:00, Alexandre Antunes 
 prof.alexandreantu...@gmail.com escreveu:


 Bom dia,

 Estou no trabalho, mas vou arriscar a minha primeira resposta no grupo.

 Desenvolvi os dois lados da expressao.

 (a/b)^2 + (b/c)^2 + (c/a)^2 = 3 + (a/b + b/c + c/a) - (b/a + c/b + a/c)

 Como (a/b)^2 = 1 + a/b - b/a
 O mesmo para os demais termos

 Fica provado a proposição.

 O que acham desse trabalhoso caminho?!?!
 Em 10/06/2015 09:00, Pacini Bores pacini.bo...@globo.com escreveu:

 Ok Mariana.

 Abraços

 Pacini

 Em 9 de junho de 2015 21:11, Mariana Groff 
 bigolingroff.mari...@gmail.com escreveu:

 Oi Pacini,
 Fiz do seguinte modo:
 f (x)=x^2-x+1/x=1 = x^3-x^2+1=x = x^3-x^2-x+1=0 =x^2
 (x-1)-(x-1)=0 = (x^2-1)(x-1)=0
 O que podemos ver que é verdade, analisando ambos os casos: em que x=1
 e o caso em que 0 x 1.
 Abraços,
 Mariana
  Em 09/06/2015 20:55, Pacini Bores pacini.bo...@globo.com escreveu:

 Oi Mariana,

 Determinei o mínimo da função usando a derivada. Não entendi o seu
 caminho, pois a função é

 f(x) = x^2-x+1/x.

 Abraços

 Pacini

 Em 9 de junho de 2015 18:09, Mariana Groff 
 bigolingroff.mari...@gmail.com escreveu:

Oi Pacini,
 Compreendi seu raciocínio. Para provar que f(x)=1, basta analisarmos
 que (x^2-1)(x-1)=0, o que verifica-se pois se x=1, o produto é 
 claramente
 não-negativo e se 0x1, vemos que, tanto x^2-1 quanto x-1 são negativos,
 tornando o produto positivo, isso?


 Em 9 de junho de 2015 11:48, Pacini Bores pacini.bo...@globo.com
 escreveu:

 Oi Mariana,
  Observe que provar  a desigualdade pedida  é equivalente  provar
 que :

 {(a/b)^2-a/b+b/a} + {(b/c)^2-b/c+c/b} +{(c/a)^2-c/a+a/c} =3, ok ?

 Agora façamos o seguinte :

 Seja f(x)= x^2-x+1/x, verifique que para x0 o valor mínimo de f é 1.

 Donde teremos a desigualdade provada.

  Estou certo pessoal ?

 Abraços

 Pacini


 Em 8 de junho de 2015 20:30, Raphael Aureliano 
 raphael0...@gmail.com escreveu:

 Ah não, desculpa, errei em Cauchy ...

 Att.
 Raphael
 Em 08/06/2015 20:27, Raphael Aureliano raphael0...@gmail.com
 escreveu:

 MA=MG
 LE=(a/b+b/c+c/a)^2=(3cbrt(abc/abc))^2 =9

 Por Cauchy
 LD=(a+b+c)(1/a+1/b+1/c)=(sqrt(a/a) +sqrt(b/b)+sqrt(c/c))^2 =9

 LE=9=LD
  Em 08/06/2015 19:20, Mariana Groff 
 bigolingroff.mari...@gmail.com escreveu:

 Boa Noite,

 (British Mathematical Olympiad - Round 2 - 2005)
 Sejam a,b e c reais positivos.
 Prove que

 (a/b+b/c+c/a)^2=(a+b+c)(1/a+1/b+1/c)

 Atenciosamente,
 Mariana

 --
 Esta mensagem foi verificada pelo sistema de antivírus e
 acredita-se estar livre de perigo.


 --
 Esta mensagem foi verificada pelo sistema de antivírus e
 acredita-se estar livre de perigo.



 --
 Esta mensagem foi verificada pelo sistema de antivírus e
 acredita-se estar livre de perigo.



 --
 Esta mensagem foi verificada pelo sistema de antivírus e
 acredita-se estar livre de perigo.



 --
 Esta mensagem foi verificada pelo sistema de antivírus e
 acredita-se estar livre de perigo.


 --
 Esta mensagem foi verificada pelo sistema de antivírus e
 acredita-se estar livre de perigo.



 --
 Esta mensagem foi verificada pelo sistema de antivírus e
 acredita-se estar livre de perigo.


 --
 Esta mensagem foi verificada pelo sistema de antivírus e
 acredita-se estar livre de perigo.



 --
 Esta mensagem foi verificada pelo sistema de antivírus e
 acredita-se estar livre de perigo.

-- 
Esta mensagem foi verificada pelo sistema de antiv�rus e
 acredita-se estar livre de perigo.



[obm-l] Re: [obm-l] Questão simples

2015-06-11 Por tôpico Pedro José
Boa tarde!

Corrigindo,

a resposta do gabarito está correta colocando o fator 10^5 para fora da
expressão, ´
q = 777*( 10^995+ 10^889+...+ 10^11 + 10^5) +77
q = 777*10^5* ( 10^990+ 10^889+...+ 10^6 + 1) +77

a última parcela será 1. Portanto o B está correto

Serão 166,  777000, seguidos da sequência  77 ou 777*B*10^5 + 77, com B
igual ao proposto no gabarito.

Saudações.


Em 11 de junho de 2015 09:58, Pedro José petroc...@gmail.com escreveu:

 Bom dia!

 O final do texto deu erro na formatação. O correto está abaixo:

 como mdc(9,1001) =1 existe 9^-1 (mod1001) onde 9^-1 ≡ 445 (mod1001)
 se 9 não dividisse, bastava multiplicar por 445 dos dois lados e a ≡ 445
 *7*(10^5-1)≡ 700 (mod1001)

 Saudações,
 PJMS

 Em 11 de junho de 2015 09:54, Pedro José petroc...@gmail.com escreveu:

 Seja um número da forma 1000..01 com n algarismos zeros, e multiplicarmos
 por um número na forma aaa.a com n+1 algarismos. Teremos como resultado
 ...a com 2*(n+1) algarismos.

 Portanto, 777 = 1001*777
 logo A = 1001*777 ( 10^995+ 10^889+...+ 10^11 + 10^5) + 7

 o resto será o resto da divisão de 7 por 1001, como 7 = 77*1001
 +700

 podemos escrever A= 1001*(777 * ( 10^995+ 10^889+...+ 10^11 + 10^5) +77)
 +700

 Como 0700 1001 == r = 700

 Já q = 777*( 10^995+ 10^889+...+ 10^11 + 10^5) +77

 Que darão 166 777000 na sequência, seguidos de três algarismos zero e
 dois algarismos 7, ou seja,

 q =77700070...7770077 166 pois (995 - 5)/6+1

 ou de outra maneira colando 777 em evidência q = 777*
 (1010...101000)*10^5 +77

 A resposta deveria ter no final do número B a seguinte sequência de
 algarismos 10 ao invés do algarismo 1 destacado em amarelo. ou então
 usar B da forma exposta e corrigir a potência de 10 em q de 5 para 8.
  q = 777*B*10^8 + 77

 Saudações,
 PJMS

 Para achar o resto dava para usar mod., mas para o quociente creio que
 não.

 A = 7 + 7*10^1 + 7*10^2 +...+ 7*10^999 + 7*10^1000

 Por soma da PG A = 7*(10^1001-1)/9
 9A  ≡ 7* (10^1001-1) (mod1001)
 como 10^6 ≡ 1 (mod 1001)

 temos que 9a ≡ 7*(10^5-1) (mod1001)

 como 9 | (10^5 -1) (| significa divide) pode-se: a ≡ 7*(1) (mod1001)
 == a ≡ 700 (mod1001)

 se 9 não dividisse, como mdc(9,1001) =1 existe 9^-1 (mod1001) onde 9^-1 ≡
 445 (mod1001)
 bastava multiplicar por 445 dos dois lados e a ≡ 445 *7*(10^5-1)≡ 700
 (mod1001)

 Em 9 de junho de 2015 22:03, marcone augusto araújo borges 
 marconeborge...@hotmail.com escreveu:

 Seja A = 777...77(1001 algarismos). Determine o quociente e o resto da
 divisão de A por 1001

 Eu achei o quociente 777000777000777000...00077 e resto 700
 o bloco 777000 reproduzido 111 vezes e mais 77 no final
 Mas o gabarito dá quociente 777.B.10^5 + 77, sendo B = 10101...1(166
  1`s )
 Não entendi  a resposta do gabarito.
 Outra coisa: daria pra achar o resto usando congruência?

 --
 Esta mensagem foi verificada pelo sistema de antivírus e
 acredita-se estar livre de perigo.





-- 
Esta mensagem foi verificada pelo sistema de antiv�rus e
 acredita-se estar livre de perigo.



[obm-l] Re: [obm-l] Função periódica

2015-06-11 Por tôpico Bernardo Freitas Paulo da Costa
2015-06-11 8:53 GMT-03:00 marcone augusto araújo borges
marconeborge...@hotmail.com:
 Seja f : R--- R definida por f(x) = sen(ax) + sen(bx), em que a e b são
 constantes reais.

 a) Se a e b são racionais, f é periódica?
Sim.

 b) Vale a recíproca do item anterior?
Não.

 Agradeço por ajuda

Sugiro que você tente mostrar o que acontece quando você soma duas
funções com períodos diferentes, digamos H e L. Depois, tente mostrar
uma condição suficiente para que a soma seja ainda periódica (com,
talvez, outro período).

Abraços,
-- 
Bernardo Freitas Paulo da Costa

-- 
Esta mensagem foi verificada pelo sistema de antiv�rus e
 acredita-se estar livre de perigo.


=
Instru��es para entrar na lista, sair da lista e usar a lista em
http://www.mat.puc-rio.br/~obmlistas/obm-l.html
=


[obm-l] Função periódica

2015-06-11 Por tôpico marcone augusto araújo borges
Seja f : R--- R definida por f(x) = sen(ax) + sen(bx), em que a e b são 
constantes reais.
a) Se a e b são racionais, f é periódica?
b) Vale a recíproca do item anterior?
Agradeço por ajuda
-- 
Esta mensagem foi verificada pelo sistema de antivírus e
 acredita-se estar livre de perigo.



[obm-l] Re: [obm-l] Questão simples

2015-06-11 Por tôpico Pedro José
Seja um número da forma 1000..01 com n algarismos zeros, e multiplicarmos
por um número na forma aaa.a com n+1 algarismos. Teremos como resultado
...a com 2*(n+1) algarismos.

Portanto, 777 = 1001*777
logo A = 1001*777 ( 10^995+ 10^889+...+ 10^11 + 10^5) + 7

o resto será o resto da divisão de 7 por 1001, como 7 = 77*1001 +700

podemos escrever A= 1001*(777 * ( 10^995+ 10^889+...+ 10^11 + 10^5) +77)
+700

Como 0700 1001 == r = 700

Já q = 777*( 10^995+ 10^889+...+ 10^11 + 10^5) +77

Que darão 166 777000 na sequência, seguidos de três algarismos zero e dois
algarismos 7, ou seja,

q =77700070...7770077 166 pois (995 - 5)/6+1

ou de outra maneira colando 777 em evidência q = 777*
(1010...101000)*10^5 +77

A resposta deveria ter no final do número B a seguinte sequência de
algarismos 10 ao invés do algarismo 1 destacado em amarelo. ou então
usar B da forma exposta e corrigir a potência de 10 em q de 5 para 8.
 q = 777*B*10^8 + 77

Saudações,
PJMS

Para achar o resto dava para usar mod., mas para o quociente creio que não.

A = 7 + 7*10^1 + 7*10^2 +...+ 7*10^999 + 7*10^1000

Por soma da PG A = 7*(10^1001-1)/9
9A  ≡ 7* (10^1001-1) (mod1001)
como 10^6 ≡ 1 (mod 1001)

temos que 9a ≡ 7*(10^5-1) (mod1001)

como 9 | (10^5 -1) (| significa divide) pode-se: a ≡ 7*(1) (mod1001)
== a ≡ 700 (mod1001)

se 9 não dividisse, como mdc(9,1001) =1 existe 9^-1 (mod1001) onde 9^-1 ≡ 445
(mod1001)
bastava multiplicar por 445 dos dois lados e a ≡ 445 *7*(10^5-1)≡ 700
(mod1001)

Em 9 de junho de 2015 22:03, marcone augusto araújo borges 
marconeborge...@hotmail.com escreveu:

 Seja A = 777...77(1001 algarismos). Determine o quociente e o resto da
 divisão de A por 1001

 Eu achei o quociente 777000777000777000...00077 e resto 700
 o bloco 777000 reproduzido 111 vezes e mais 77 no final
 Mas o gabarito dá quociente 777.B.10^5 + 77, sendo B = 10101...1(166
  1`s )
 Não entendi  a resposta do gabarito.
 Outra coisa: daria pra achar o resto usando congruência?

 --
 Esta mensagem foi verificada pelo sistema de antivírus e
 acredita-se estar livre de perigo.


-- 
Esta mensagem foi verificada pelo sistema de antiv�rus e
 acredita-se estar livre de perigo.



[obm-l] Re: [obm-l] Questão simples

2015-06-11 Por tôpico Pedro José
Bom dia!

O final do texto deu erro na formatação. O correto está abaixo:

como mdc(9,1001) =1 existe 9^-1 (mod1001) onde 9^-1 ≡ 445 (mod1001)
se 9 não dividisse, bastava multiplicar por 445 dos dois lados e a ≡ 445
*7*(10^5-1)≡ 700 (mod1001)

Saudações,
PJMS

Em 11 de junho de 2015 09:54, Pedro José petroc...@gmail.com escreveu:

 Seja um número da forma 1000..01 com n algarismos zeros, e multiplicarmos
 por um número na forma aaa.a com n+1 algarismos. Teremos como resultado
 ...a com 2*(n+1) algarismos.

 Portanto, 777 = 1001*777
 logo A = 1001*777 ( 10^995+ 10^889+...+ 10^11 + 10^5) + 7

 o resto será o resto da divisão de 7 por 1001, como 7 = 77*1001
 +700

 podemos escrever A= 1001*(777 * ( 10^995+ 10^889+...+ 10^11 + 10^5) +77)
 +700

 Como 0700 1001 == r = 700

 Já q = 777*( 10^995+ 10^889+...+ 10^11 + 10^5) +77

 Que darão 166 777000 na sequência, seguidos de três algarismos zero e dois
 algarismos 7, ou seja,

 q =77700070...7770077 166 pois (995 - 5)/6+1

 ou de outra maneira colando 777 em evidência q = 777*
 (1010...101000)*10^5 +77

 A resposta deveria ter no final do número B a seguinte sequência de
 algarismos 10 ao invés do algarismo 1 destacado em amarelo. ou então
 usar B da forma exposta e corrigir a potência de 10 em q de 5 para 8.
  q = 777*B*10^8 + 77

 Saudações,
 PJMS

 Para achar o resto dava para usar mod., mas para o quociente creio que não.

 A = 7 + 7*10^1 + 7*10^2 +...+ 7*10^999 + 7*10^1000

 Por soma da PG A = 7*(10^1001-1)/9
 9A  ≡ 7* (10^1001-1) (mod1001)
 como 10^6 ≡ 1 (mod 1001)

 temos que 9a ≡ 7*(10^5-1) (mod1001)

 como 9 | (10^5 -1) (| significa divide) pode-se: a ≡ 7*(1) (mod1001)
 == a ≡ 700 (mod1001)

 se 9 não dividisse, como mdc(9,1001) =1 existe 9^-1 (mod1001) onde 9^-1 ≡ 445
 (mod1001)
 bastava multiplicar por 445 dos dois lados e a ≡ 445 *7*(10^5-1)≡ 700
 (mod1001)

 Em 9 de junho de 2015 22:03, marcone augusto araújo borges 
 marconeborge...@hotmail.com escreveu:

 Seja A = 777...77(1001 algarismos). Determine o quociente e o resto da
 divisão de A por 1001

 Eu achei o quociente 777000777000777000...00077 e resto 700
 o bloco 777000 reproduzido 111 vezes e mais 77 no final
 Mas o gabarito dá quociente 777.B.10^5 + 77, sendo B = 10101...1(166
  1`s )
 Não entendi  a resposta do gabarito.
 Outra coisa: daria pra achar o resto usando congruência?

 --
 Esta mensagem foi verificada pelo sistema de antivírus e
 acredita-se estar livre de perigo.




-- 
Esta mensagem foi verificada pelo sistema de antiv�rus e
 acredita-se estar livre de perigo.